GMAT Exam pack 2 - CR

This topic has expert replies
Senior | Next Rank: 100 Posts
Posts: 97
Joined: Mon Aug 04, 2014 5:39 pm
Thanked: 1 times
Followed by:2 members

GMAT Exam pack 2 - CR

by lucas211 » Wed Jun 22, 2016 2:15 am
Hello BTG

Would appreciate som explanation to finding the right answer in the following question.

Thanks in advance.
Attachments
Screen Shot 2016-06-22 at 12.13.26.png

User avatar
GMAT Instructor
Posts: 15539
Joined: Tue May 25, 2010 12:04 pm
Location: New York, NY
Thanked: 13060 times
Followed by:1906 members
GMAT Score:790

by GMATGuruNY » Wed Jun 22, 2016 3:30 am
Nalmed Province's plan is to reduce highway congestion by expanding the commuter rail system, so giving more people the option of traveling by train. When a recent opinion poll presented this plan to province residents, they overwhelmingly favored it, even though they knew that enacting the plan would mean substantial tax increases. Consequently, the plan, if enacted, is very likely to succeed, because if people are prepared to pay, they expect to reap the benefit.

The inference made from the poll results is most vulnerable to criticism on the grounds that

it overlooks the possibility that once the highways are consistently uncongested, some commuters might have no motivation to switch from using their car to using the rail system

the favorable responses collected are entirely consistent with every one of those respondents expecting that it would be others who, by using the rail system, would ease highway congestion

those respondents who opposed the plan might nevertheless become users of the rail system as a result of its expansion

those respondents who opposed the plan might oppose it for reasons other than the tax increase required to carry it out

residents responding to the poll are likely to overestimate the tax increase they themselves will experience if the proposed expansion occurs
Premise: When a recent opinion poll presented the plan to province residents, they overwhelmingly favored it.
Conclusion: The plan to expand the rail system will succeed in reducing highway congestion.

This argument links the poll results to a reduction in highway congestion.
It assumes that the people in the poll will actually USE the rail system and thereby ease highway congestion.
One way to weaken the argument is to attack this assumption.

B: the favorable responses collected are entirely consistent with every one of those respondents expecting that it would be others who, by using the rail system, would ease highway congestion
Here, the people in the poll expect that OTHERS will use the rail system, invalidating the assumption that the PEOPLE IN THE POLL will use the rail system and thereby ease highway congestion.

The correct answer is B.
Private tutor exclusively for the GMAT and GRE, with over 20 years of experience.
Followed here and elsewhere by over 1900 test-takers.
I have worked with students based in the US, Australia, Taiwan, China, Tajikistan, Kuwait, Saudi Arabia -- a long list of countries.
My students have been admitted to HBS, CBS, Tuck, Yale, Stern, Fuqua -- a long list of top programs.

As a tutor, I don't simply teach you how I would approach problems.
I unlock the best way for YOU to solve problems.

For more information, please email me (Mitch Hunt) at [email protected].
Student Review #1
Student Review #2
Student Review #3

Junior | Next Rank: 30 Posts
Posts: 11
Joined: Tue Jul 12, 2016 1:15 pm

by Phoenix7 » Tue Jul 12, 2016 2:00 pm
The choice is between options A and B. Both , if true, will weaken the argument. B is the stronger choice because it involves every one of those respondents whereas A involves only some of them.

Senior | Next Rank: 100 Posts
Posts: 39
Joined: Sat Jan 12, 2019 1:08 am

by meenakshimiyer » Sat Jan 12, 2019 8:34 pm
A. This statement claims that once the highways are uncongested, some people might not switch to rail transport. The plan will succeed even if this statement is true, as people apart from those who won't switch, would have the motivation to use the rail system thus reducing the congestion.
B. If each one expects others to use the train (and not themselves); they essentially negate each other's intentions, and hence no one uses the train. Hence, B is the answer.
C. This would rather strengthen the conclusion made in the argument.
D. This statement unnecessarily expands the scope of argument providing no substantial criticism to the conclusion.
E. The statement strengthens the conclusion made in the argument.